LSAT Q&A: Ask the Experts Forum

Prepare for the LSAT or discuss it with others in this forum.
Post Reply
MLBrandow

Bronze
Posts: 121
Joined: Wed Mar 15, 2006 5:12 pm

Re: LSAT Q&A: Ask the Experts

Post by MLBrandow » Thu May 31, 2012 11:23 pm

Just added another expert for your questioning pleasure: SanDiegoJake, from the Princeton Review.

Thanks for offering your time to help everyone out, and a big thanks again to all the other experts who are giving so much great advice! :)

Manhattan LSAT Noah

Silver
Posts: 744
Joined: Tue Oct 12, 2010 8:43 am

Re: LSAT Q&A: Ask the Experts

Post by Manhattan LSAT Noah » Thu May 31, 2012 11:31 pm

dkb17xzx wrote:Thanks for doing this experts.

PT 61 Section 4, Q. 21 - It came down to C and D, and I picked D (incorrect). I notice that there is a shift from modern mammals to prehistoric marine reptiles. Is C a better answer because it bridges this and says that of both modern and prehistoric marine reptiles that were not deep divers, most had porous bones. So just b/c a prehistoric marine reptile has the same characteristic as a modern mammal, it doesn't mean it was able to do the same thing (deep dive) BECAUSE most this characteristic is common to the reptiles who are not deep divers?

I feel that D still weakens - but not as much as C.
I don't think you're getting the core on this--the jump from prehistoric to modern isn't really an issue here (good thinking, but there's also no reason to think the physical facts of today don't apply to prehistoric animals). Here's an analogy to help you see what the issue is with this one:

law professors have elbow patches on their jackets -- this allows them to gesture a lot without ripping holes in their jackets.

Noah has elbow patches on his jackets, so he's probably a law professor.

In the argument at hand, the conclusion is that ich were deep divers. Why? Because they have the same sort of bone structure that modern deep divers have, which allows deep diving.

What's the gap? Well, maybe they have that structure, but they have it for some other reason. Maybe the porous bone structure also helps you run quickly.

(C) plays on this gap. If animals that didn't deep dive have that bone structure, then is it really an indication of deep-diving? No.

(A) is about species that do not have porous outer shells. Out of scope. Similarly, who cares if you have to surface or not?

(B) is about animals without the porous bone structure. Who cares about them?

(D) is tempting, however the big indication that this answer is fishy :lol: is that "there is no clear evidence" -- that leaves a lot of wiggle room. Just because there isn't evidence, doesn't mean the phenomenon doesn't exist.

(E) is tempting as well. However, it's simply saying that the ich probably didn't even need porous bones. But so what? Does that mean that them having them indicates they were deep divers?

I hope that clears it up.

Manhattan LSAT Noah

Silver
Posts: 744
Joined: Tue Oct 12, 2010 8:43 am

Re: LSAT Q&A: Ask the Experts

Post by Manhattan LSAT Noah » Thu May 31, 2012 11:39 pm

dkb17xzx wrote:Also, PT 61, Section 4, # 24. Does "likely" affect the way a conditional reasoning is diagrammed?
Yes, you have to remember that all that is guaranteed is a likelihood. But, your diagram looks fine.

However, I get the sense you're over-relying on diagramming here. You sum it up nicely here:
dkb17xzx wrote:the argument never addresses whether someone feels comfortable approaching a stranger not of one's age
In short, the argument says that you feel comfortable approach strangers your own age, and then goes on to draw a conclusion that acts as is you don't feel comfortable approach strangers that are older/younger. So, you can't make a conclusion about comfort that includes the assumption that comfort means the same age.

I hope that's not stupidly oversimplifying the situation. But, from your post, it seemed like maybe that's what would unlock it for you. A much longer discussion of the question is here.

Manhattan LSAT Noah

Silver
Posts: 744
Joined: Tue Oct 12, 2010 8:43 am

Re: LSAT Q&A: Ask the Experts

Post by Manhattan LSAT Noah » Thu May 31, 2012 11:48 pm

cc.celina wrote:
Manhattan LSAT Noah wrote: I hope that helps. There's a longer discussion of this question on our forums: http://www.manhattanlsat.com/forums/post2163.html
Thanks a lot, your explanation and the discussion there really helped clear that up for me. For some reason i wasn't understanding that distinction between "greatly increased" and just plain "great."

Maybe this question is along similar lines: PT 32, Sec 1, Q 12 (polar bears).
I picked E, because I thought that if polar bears could smell familiar territory, that would go against one of the conditions necessary for it to qualify as navigation: it has to be "beyond the immediate range of the animal's senses." If the bear can smell the familiar territory, the territory isn't beyond the range of their senses.
I understand that B goes against the other, that it goes from "unfamiliar territory" to "points familiar to the animal."

I've been told that the right answer is 100% right and the other four are its logical opposite, and 100% wrong. Is E wrong because even though polar bears could smell familiar territory, it wasn't within the "immediate" range of their senses, just within plain old range?
Glad to help. (I've been told by MLBrandow that if I answer 100 questions I get a free drink ticket or something.)

In the polar bear question, (E) is off because the bear in the stimulus wasn't released in familiar territory. It was released away from its home territory. We can argue that perhaps those bears are familiar with 500 km of territory, but we can just as easily argue that they aren't.

As for the four wrong answers being the logical opposite of the right answer, I don't really think that way. The right answer is right, and the others are not. I guess that's the logical opposite! But, I think the way you're saying it makes it seem as if the 4 wrong answers will damage the argument. Not true (except in a weaken except question). I agree that the 4 wrong answers are 100% wrong. I think you're right to dig deeper into wrong answer analysis.

humbugger

New
Posts: 47
Joined: Tue Mar 27, 2012 10:08 pm

Re: LSAT Q&A: Ask the Experts

Post by humbugger » Sat Jun 02, 2012 5:37 pm

62.2.19

I thought I had a pretty good handle on these formal logic questions, but this one exposed some weakness of mine:

The stimulus seems straightforward:

TF and ~EB -> G -> ~IO

I wrote the contrapositive as

IO -> ~TF or EB

Which looks a lot like (E) (an incorrect answer).

All of my gratitude to anyone who can tell me what I'm doing wrong!

Want to continue reading?

Register now to search topics and post comments!

Absolutely FREE!


User avatar
shifty_eyed

Gold
Posts: 1925
Joined: Fri Oct 02, 2009 8:09 pm

Re: LSAT Q&A: Ask the Experts

Post by shifty_eyed » Sat Jun 02, 2012 5:55 pm

humbugger wrote:62.2.19

I thought I had a pretty good handle on these formal logic questions, but this one exposed some weakness of mine:

The stimulus seems straightforward:

TF and ~EB -> G -> ~IO

I wrote the contrapositive as

IO -> ~TF or EB

Which looks a lot like (E) (an incorrect answer).

All of my gratitude to anyone who can tell me what I'm doing wrong!
I'm no expert, but I just took PT 62 and got this one right. E is wrong bc it means that an independently owned pet store HAS to sell exotic birds if it doesn't sell tropical fish. An IO could sell neither TF or EB. I'm not great at formal logic and I didn't diagram this one, but I think this could be written as IO -> ~(TF and ~EB) maybe? The takeaway is if it's independently owned, it doesn't sell EB and not TF.

deepblue

New
Posts: 40
Joined: Tue Oct 04, 2011 5:42 pm

Re: LSAT Q&A: Ask the Experts

Post by deepblue » Sat Jun 02, 2012 6:26 pm

Do you have any tips for picking between two very similar RC answers?

BalanceCare

Bronze
Posts: 116
Joined: Thu Feb 09, 2012 7:56 pm

Re: LSAT Q&A: Ask the Experts

Post by BalanceCare » Sat Jun 02, 2012 7:42 pm

Manhattan LSAT Noah wrote:
BalanceCare wrote: PT63, LR1 #10:

I picked the right answer, but something about it seems weird. (I think this is similar to a question that was recently asked but I’m still having trouble with the concept so please excuse the repetition.)
It seems that the argument consists of:
P: [Ms. S’s actions led to damage]
A: [If you could reasonably expect that your action would cause damage] --> [Pay for the damage]
C: [If Ms. S could reasonably expect that action would lead to damage] --> [She should pay for the damage]
As I see it, the conclusion is itself a conditional principal, and this seems odd. It seems like we should be trying to find out whether or not Ms. S had a reasonable expectation of the damage or not, as this will determine whether or not she should pay. I see that A, the correct answer, connects the premise to the conclusion, but the whole thing just seems weird to me. Can anyone illuminate this?
You're right that the conclusion is a relationship. Usually we see this in sufficient assumption questions, but hey, here we are.

You have the argument structure down nicely. And, this is another principle support question, so expect the answer to be the assumption. As to your point about what it seems like we should be trying to find, we don't need to prove that the conditional conclusion gets triggered, we just need to show that it is logically valid.

Similarly, if I asked you to support this: If people stop smoking they live a month longer. You don't need to support whether people actually stop smoking. You just need to show that if they do, they'll live a month longer.

I hope that clears it up.
Thank you for that, Noah.

So, basically, is it fair to say that if a sufficient assumption question (like this one) asks us to justify a conclusion that is itself a conditional statement, then the right answer will be a conditional statement almost identical to the conclusion in a general sense, but without the facts specific to the situation described in the stimulus?

(Which is how I read the above Q from PT63:

P: [Ms. S’s actions led to damage]
C: [If Ms. S could reasonably expect that action would lead to damage] --> [She should pay for the damage][/quote]

A: [If you could reasonably expect that your action would cause damage] --> [Pay for the damage]


If that is the case, then this question's stupidity / simplicity is what makes it difficult ... it's like a zen koan or something...

humbugger

New
Posts: 47
Joined: Tue Mar 27, 2012 10:08 pm

Re: LSAT Q&A: Ask the Experts

Post by humbugger » Sun Jun 03, 2012 3:14 am

shifty_eyed wrote: I'm no expert, but I just took PT 62 and got this one right. E is wrong bc it means that an independently owned pet store HAS to sell exotic birds if it doesn't sell tropical fish. An IO could sell neither TF or EB. I'm not great at formal logic and I didn't diagram this one, but I think this could be written as IO -> ~(TF and ~EB) maybe? The takeaway is if it's independently owned, it doesn't sell EB and not TF.
Thanks for replying. I believe (E), if it were true, should be diagrammed IO -> TF or EB, since, in that case, an IO that doesn't sell TF must then sell EB to meet the necessary condition.

Anyway, I figured out what I was doing wrong: Essentially, IO -> ~TF or EB means that if an IO does not sell TF then it already meets the "or" condition, and so it could or could not sell EB. Now that I've realized it, I feel kind of dumb, actually.

Want to continue reading?

Register for access!

Did I mention it was FREE ?


Manhattan LSAT Noah

Silver
Posts: 744
Joined: Tue Oct 12, 2010 8:43 am

Re: LSAT Q&A: Ask the Experts

Post by Manhattan LSAT Noah » Mon Jun 04, 2012 11:59 am

BalanceCare wrote:If that is the case, then this question's stupidity / simplicity is what makes it difficult ... it's like a zen koan or something...
This question is rather annoying like that. Similarly, I hate those questions where the premise is something like "If Jim is correct..." and then the conclusion acts as if he is, and the answer is that he is actually correct.

I'll have to think about whether I'd create a rule like you just did, but definitely a pattern to be aware of.

helpplease

Bronze
Posts: 130
Joined: Fri Jun 01, 2012 2:12 pm

Re: LSAT Q&A: Ask the Experts

Post by helpplease » Mon Jun 04, 2012 10:32 pm

Could you all please provide some insight into PT10, Section 4, #24. I do not understand why C is correct.

It tells us that the difference between the rate of inflation and the rate of return on the most profitable invest. is the % (at a minimum???) by which the value of ANY investment will decline. So if the value of an investment declines MORE than that percentage why does this investment have to be less profitable than the most profitable investment available? Couldn't the most profitable investment available decrease by more than the minimum as well?

I know i'm not comprehending something fully here so any help would be greatly appreciated. Thank you!!!

Manhattan LSAT Noah

Silver
Posts: 744
Joined: Tue Oct 12, 2010 8:43 am

Re: LSAT Q&A: Ask the Experts

Post by Manhattan LSAT Noah » Tue Jun 05, 2012 1:35 pm

helpplease wrote:Could you all please provide some insight into PT10, Section 4, #24. I do not understand why C is correct.

It tells us that the difference between the rate of inflation and the rate of return on the most profitable invest. is the % (at a minimum???) by which the value of ANY investment will decline. So if the value of an investment declines MORE than that percentage why does this investment have to be less profitable than the most profitable investment available? Couldn't the most profitable investment available decrease by more than the minimum as well?

I know i'm not comprehending something fully here so any help would be greatly appreciated. Thank you!!!
I think you can figure it out with this: what is used to set the decline rate ?

Answer (highlight): The decline rate (difference between inflation and the rate of return) is set using the most profitable investment around. So, when we talk about an investment that is declining earning less than that is not that most profitable investment and must be less profitable.

That make sense? If not, there's a longer discussion here.

bp shinners

Gold
Posts: 3086
Joined: Wed Mar 16, 2011 7:05 pm

Re: LSAT Q&A: Ask the Experts

Post by bp shinners » Tue Jun 05, 2012 1:38 pm

helpplease wrote:Could you all please provide some insight into PT10, Section 4, #24. I do not understand why C is correct.

It tells us that the difference between the rate of inflation and the rate of return on the most profitable invest. is the % (at a minimum???) by which the value of ANY investment will decline. So if the value of an investment declines MORE than that percentage why does this investment have to be less profitable than the most profitable investment available? Couldn't the most profitable investment available decrease by more than the minimum as well?

I know i'm not comprehending something fully here so any help would be greatly appreciated. Thank you!!!
So this is one of those mathy ones that distracts people from what it's saying by trying to make you freak out because of the math.

I've got a few ideas here:
Rate of inflation - just the rate of inflation
Rate of return on the most profitable investment available - this is like saying the best/highest interest you can receive when investing
When inflation is going up faster than what I'm earning on THE MOST profitable investment, I'm essentially losing money, because my money's going down in value faster than I'm accumulating it. The difference that the stimulus talks about here is the difference between the rate of inflation and THE MOST profitable investment. Since it's THE MOST profitable, I'm making the most return on my investment with it. Therefore, anything else I'm investing in will provide a lower rate of return, and thus the difference between inflation and that rate of return (the less-than-the-most rate of return) will be greater.

So let's say that inflation is 4%. The investment that is THE MOST profitable is 2%. The difference here is 2%, so I'm losing that much on my investment. When I look at an investment that declines in value by more than than percentage (by more than the 2% above, the difference between the two), I would be looking at something that, for instance, is losing 3%. If I want the difference between the rate of inflation and the return on my investment to be 3%, the rate of return on that investment must be 1%. 1% is less than 2%, so answer C is correct.

Interestingly, C is something that just generally must be true.

Now, to your contention. The most profitable investment available could not decrease by more than the minimum as well. Why not? Because I'm using the most profitable investment to define the minimum. The minimum here is defined as the difference between the rate of inflation and the rate of return on the most profitable investment. As such, those two concepts are tied together.

Register now!

Resources to assist law school applicants, students & graduates.

It's still FREE!


slystad

New
Posts: 6
Joined: Sun May 20, 2012 12:54 am

Re: LSAT Q&A: Ask the Experts

Post by slystad » Wed Jun 06, 2012 10:37 am

Hello experts,

Thanks very much for doing this. I came across PT56.S3.#24. For some reason, none of the answers looked even slightly appealing to me, though B was the least worst, in that, given the shifting music tastes, maybe it would cycle around to the local radio station? Which was a bad guess, but for some reason, these answers look like they belonged to a different question. Could you help me out a bit with them?

Manhattan LSAT Noah

Silver
Posts: 744
Joined: Tue Oct 12, 2010 8:43 am

Re: LSAT Q&A: Ask the Experts

Post by Manhattan LSAT Noah » Wed Jun 06, 2012 11:31 am

slystad wrote:Hello experts,

Thanks very much for doing this. I came across PT56.S3.#24. For some reason, none of the answers looked even slightly appealing to me, though B was the least worst, in that, given the shifting music tastes, maybe it would cycle around to the local radio station? Which was a bad guess, but for some reason, these answers look like they belonged to a different question. Could you help me out a bit with them?
I see what you mean! You're looking for "there's a history of X and therefore X will probably keep happening since nothing has been done to stop X and some things have been done to continue it." However, we definitely don't get that! I worked wrong-to-right on this one:

(A) Every is too strong. Eliminate.
(B) seems good. Defer.
(C) All too strong. Eliminate.
(D) seems possible. Defer.
(E) Only too strong. Eliminate.

Down to (B) and (D). Here we want to look for conclusion mismatches. We want to here a conclusion about something definitely occurring/not occurring. (B) has a "probably" and (D) has a "will be" -- so (B) is out and (D) works. Let's confirm: we want a firm prediction based on a history of something, and (D) has it.

It's missing all the extra support, but it's is the most similar.

Probably what you want to work on is seeking out conclusion mismatches, and then premise mismatches.

I hope that helps. There's a longer discussion here, discussing some other issues with the answers choices.

1278

New
Posts: 34
Joined: Thu Jun 07, 2012 4:26 pm

Re: LSAT Q&A: Ask the Experts

Post by 1278 » Thu Jun 07, 2012 4:38 pm

Dear Experts,

I have been reviewing RC for the Monday test and realize that I still have quite a weakness for the main point questions.

For PT45 RC Q1, I picked A because I thought D is only talking about problems but A kind of gives a perspective, which will be more in line with the passage. Of course I was wrong. Is it because A says "the most useful" and this is too strong? If it says "a more useful response", would it be correct?

For PT45 RC Q15, I also had E because I thought it has "more stuff" init than D. I don't think E is wrong --- the fact that they are placed in the same tree DOES indicate their common evolutionary origin, that's the point of having these trees. so I totally don't understand why D is better.

PT42 RC Q1, same problem: why is B better than D?

Also, if you can give me some general advices on main point questions, that would be really great!!

thanks a ton

User avatar
Dave Hall

Bronze
Posts: 186
Joined: Thu Jul 28, 2011 4:18 pm

Re: LSAT Q&A: Ask the Experts

Post by Dave Hall » Thu Jun 07, 2012 6:40 pm

1278 wrote:Dear Experts,

I have been reviewing RC for the Monday test and realize that I still have quite a weakness for the main point questions.

For PT45 RC Q1, I picked A because I thought D is only talking about problems but A kind of gives a perspective, which will be more in line with the passage. Of course I was wrong. Is it because A says "the most useful" and this is too strong? If it says "a more useful response", would it be correct?
You've identified what's probably the most useful (most useful because it's the most replicable) part of the problem with (A). Notice, too, that (A) says we'll forego all immediate aid in favor of long-term - that can't be right, can it?
1278 wrote:For PT45 RC Q15, I also had E because I thought it has "more stuff" init than D. I don't think E is wrong --- the fact that they are placed in the same tree DOES indicate their common evolutionary origin, that's the point of having these trees. so I totally don't understand why D is better.
So similar! Aaargh! Eyes... melting!

So let's consider their differences - (D) says the point is that we've uncovered the relationship, where (E) says the point is that we've explained the visible similarities. While (E) is certainly true, it's only one part of what's important about the new research - (D) captures the broader implications, in the same way that the passage is concerned with the relationship more broadly.
1278 wrote:PT42 RC Q1, same problem: why is B better than D?
Again, our work is to identify the differences - here, (B) - like the passage - tells us he innovated the techniques, where (D) says he only adopted previously existing methods. Again, these are very similar, but by focusing on the small things that separate them, we can more clearly see why one is correct.
1278 wrote:Also, if you can give me some general advices on main point questions, that would be really great!!

thanks a ton
Based only on these three examples, I'd say your problem is not with identifying the main point, per se, but is instead a difficulty choosing an answer choice. The most important thing is to have an arbiter - you need a way of deciding what the right answer should look like. You accomplish this while you're reading, by keeping a literally geographic focus on finding a sentence within the passage that most clearly expresses the author's point. Such a sentence is in there - find it, and you're basically home. After that, adopt the comparison technique discussed above, and you should find you meet with greater success.

Get unlimited access to all forums and topics

Register now!

I'm pretty sure I told you it's FREE...


1278

New
Posts: 34
Joined: Thu Jun 07, 2012 4:26 pm

Re: LSAT Q&A: Ask the Experts

Post by 1278 » Thu Jun 07, 2012 10:15 pm

Thanks so much Dave for the quick and very helpful response! Really appreciate it.

slystad

New
Posts: 6
Joined: Sun May 20, 2012 12:54 am

Re: LSAT Q&A: Ask the Experts

Post by slystad » Fri Jun 08, 2012 6:52 am

Thanks Noah! I was looking way too hard at it, wish I had realized it was that simple.

Manhattan LSAT Noah

Silver
Posts: 744
Joined: Tue Oct 12, 2010 8:43 am

Re: LSAT Q&A: Ask the Experts

Post by Manhattan LSAT Noah » Fri Jun 08, 2012 10:26 am

slystad wrote:Thanks Noah! I was looking way too hard at it, wish I had realized it was that simple.
Great! Match the reasoning/flaw questions are the ones where most people wipe out or lose a lot of time because they don't have an approach, they just use intuition. Great place to gain an advantage.

dkb17xzx

Bronze
Posts: 403
Joined: Wed Jan 20, 2010 6:25 pm

Re: LSAT Q&A: Ask the Experts

Post by dkb17xzx » Fri Jun 08, 2012 10:34 am

Noah,

Thank you for your help before. The simplification certainly helped. As much as I dislike studying for the LSAT, I have to admit that the people who write the questions are geniuses.

Communicate now with those who not only know what a legal education is, but can offer you worthy advice and commentary as you complete the three most educational, yet challenging years of your law related post graduate life.

Register now, it's still FREE!


Manhattan LSAT Noah

Silver
Posts: 744
Joined: Tue Oct 12, 2010 8:43 am

Re: LSAT Q&A: Ask the Experts

Post by Manhattan LSAT Noah » Fri Jun 08, 2012 11:16 am

dkb17xzx wrote:Noah,

Thank you for your help before. The simplification certainly helped. As much as I dislike studying for the LSAT, I have to admit that the people who write the questions are geniuses.
I agree. As someone that teaches this stuff (and has taught GMAT), I'd say the LSAT is a really well-written test.

MLBrandow

Bronze
Posts: 121
Joined: Wed Mar 15, 2006 5:12 pm

Re: LSAT Q&A: Ask the Experts

Post by MLBrandow » Sat Jun 09, 2012 10:45 am

64.3.17

In reviewing PT64, out of 13 mistakes (shameful, I know), this one stood out as feeling singly like some kind of comprehension or general strategy error for me.

I see the connection and the answer now, but with all the terms and concepts floating around here, I just got bogged down and lost focus. I actually had to skip and return to this problem after doing a few others, and ended up never liking any of the answers I was between (a sure sign of a miss).

How should I be approaching this problem? Should a flag have been raised because anti-inflammatory drug treatment is just kind of thrown in there?

Thanks for your help!

1278

New
Posts: 34
Joined: Thu Jun 07, 2012 4:26 pm

Re: LSAT Q&A: Ask the Experts

Post by 1278 » Sat Jun 09, 2012 3:04 pm

Hi Experts,

PT65 S4 Q5:
I felt very "WTF" when seeing this question. B is kind of obvious; but although it supports the conclusion, it makes 95% of the entire argument (codefendants/shared counsel/blabla) not necessary. Is this considered "strongly supporting" the argument? should I be prepared for a similar TMI stimulus on Monday??

Also, what do the experts think about the RC section in PT65? I TOTALLY screwed up, could not understand the blackmail passage AT ALL (didn't realize that part B indicates that blackmailing per se is not illegal). Wasted way too much time on this one and didn't have enough time to finish the last passage. A big lesson on time management for me. But really, I typically scored RC between -1 and -5 and NEVER had such a time issue. After the easy RC of PT64 (-1), this was a huge shock.

thanks so much!

User avatar
Dave Hall

Bronze
Posts: 186
Joined: Thu Jul 28, 2011 4:18 pm

Re: LSAT Q&A: Ask the Experts

Post by Dave Hall » Sat Jun 09, 2012 8:24 pm

MLBrandow wrote:64.3.17

In reviewing PT64, out of 13 mistakes (shameful, I know), this one stood out as feeling singly like some kind of comprehension or general strategy error for me.

I see the connection and the answer now, but with all the terms and concepts floating around here, I just got bogged down and lost focus. I actually had to skip and return to this problem after doing a few others, and ended up never liking any of the answers I was between (a sure sign of a miss).

How should I be approaching this problem? Should a flag have been raised because anti-inflammatory drug treatment is just kind of thrown in there?

Thanks for your help!
I'm taking a structural view here; the writer claims that it's caused by thing X because it can be treated by thing Y. Now I'm asking myself what the hell Y has to do with X. The author has assumed there's a connection, but hasn't given any evidence of it.

This is another way of saying what you've said above (emphasis added). So breathe; you've got this.

Seriously? What are you waiting for?

Now there's a charge.
Just kidding ... it's still FREE!


Post Reply

Return to “LSAT Prep and Discussion Forum”